Let $A$ be an infinite set and B a countable set, show that $vert A cup B vert = vert A vert$ [duplicate] Announcing the arrival of Valued Associate #679: Cesar Manara Planned maintenance scheduled April 17/18, 2019 at 00:00UTC (8:00pm US/Eastern)Cardinality of the union of infinite and countable setsLet the set S be infinite, and the set T countably infinite. Show that S and S U T have the same cardinalitySize of infinte sets cardinalityIs this proof for $A,B$ countable $implies Acup B$ countable?Trouble with definition of countable, denumerableProperties of Infinite set on co-finite topology and Countable set on co-countable topologyIf A is an infinite set and B is at most countable set, prove that A and $A cup B$ have the same cardinalityProve that if $A$ is any infinite set, the set of all finite subsets of $A$ has the same cardinality as $A$Show that if $ A cup B = A$ and $ Acap B = A$ then $ A = B$Suppose that $A$ is finite and that $f:A to B$ is surjective. Then $B$ is finite and $vertBvert leq vertAvert$Cardinality of the set of infinite binary sequencesPartition an infinite set into countable setsIf $C$ is infinite and $B$ is finite, then $Csetminus B$ is infinite.

Is high blood pressure ever a symptom attributable solely to dehydration?

When to stop saving and start investing?

What's the purpose of writing one's academic bio in 3rd person?

What is the musical term for a note that continously plays through a melody?

Storing hydrofluoric acid before the invention of plastics

Single word antonym of "flightless"

Antler Helmet: Can it work?

Can a non-EU citizen traveling with me come with me through the EU passport line?

List *all* the tuples!

Why don't the Weasley twins use magic outside of school if the Trace can only find the location of spells cast?

Should I call the interviewer directly, if HR aren't responding?

Bonus calculation: Am I making a mountain out of a molehill?

Is there a concise way to say "all of the X, one of each"?

What is the longest distance a 13th-level monk can jump while attacking on the same turn?

What does the "x" in "x86" represent?

Does surprise arrest existing movement?

What happens to sewage if there is no river near by?

Is 1 ppb equal to 1 μg/kg?

Why does Python start at index 1 when iterating an array backwards?

Letter Boxed validator

How do I mention the quality of my school without bragging

How can I fade player when goes inside or outside of the area?

What is a Meta algorithm?

Can inflation occur in a positive-sum game currency system such as the Stack Exchange reputation system?



Let $A$ be an infinite set and B a countable set, show that $vert A cup B vert = vert A vert$ [duplicate]



Announcing the arrival of Valued Associate #679: Cesar Manara
Planned maintenance scheduled April 17/18, 2019 at 00:00UTC (8:00pm US/Eastern)Cardinality of the union of infinite and countable setsLet the set S be infinite, and the set T countably infinite. Show that S and S U T have the same cardinalitySize of infinte sets cardinalityIs this proof for $A,B$ countable $implies Acup B$ countable?Trouble with definition of countable, denumerableProperties of Infinite set on co-finite topology and Countable set on co-countable topologyIf A is an infinite set and B is at most countable set, prove that A and $A cup B$ have the same cardinalityProve that if $A$ is any infinite set, the set of all finite subsets of $A$ has the same cardinality as $A$Show that if $ A cup B = A$ and $ Acap B = A$ then $ A = B$Suppose that $A$ is finite and that $f:A to B$ is surjective. Then $B$ is finite and $vertBvert leq vertAvert$Cardinality of the set of infinite binary sequencesPartition an infinite set into countable setsIf $C$ is infinite and $B$ is finite, then $Csetminus B$ is infinite.










0












$begingroup$



This question already has an answer here:



  • Cardinality of the union of infinite and countable sets

    2 answers



  • Let the set S be infinite, and the set T countably infinite. Show that S and S U T have the same cardinality

    1 answer



Can someone please verify if my proof is correct, and add to it if it isn't?



Since we're given that $B$ is countable, we know $B$ is either finite or has the same cardinality as, say, the set $mathbbQ$. So $vert B vert = aleph_0$.



Similarly, if $A$ is infinite, then we know the result $aleph_0 leq vert A vert$.



This implies that $ vert B vert leq vert A vert$.



Then $vert A cup B vert = vert A vert$.



Am I missing something here?










share|cite|improve this question











$endgroup$



marked as duplicate by Asaf Karagila elementary-set-theory
Users with the  elementary-set-theory badge can single-handedly close elementary-set-theory questions as duplicates and reopen them as needed.

StackExchange.ready(function()
if (StackExchange.options.isMobile) return;

$('.dupe-hammer-message-hover:not(.hover-bound)').each(function()
var $hover = $(this).addClass('hover-bound'),
$msg = $hover.siblings('.dupe-hammer-message');

$hover.hover(
function()
$hover.showInfoMessage('',
messageElement: $msg.clone().show(),
transient: false,
position: my: 'bottom left', at: 'top center', offsetTop: -7 ,
dismissable: false,
relativeToBody: true
);
,
function()
StackExchange.helpers.removeMessages();

);
);
);
Mar 27 at 10:50


This question has been asked before and already has an answer. If those answers do not fully address your question, please ask a new question.

















  • $begingroup$
    Please include the original question in the body of your question.
    $endgroup$
    – coreyman317
    Mar 26 at 7:51










  • $begingroup$
    The answer to this question will be practically the same as the answer to math.stackexchange.com/questions/3162848/… .
    $endgroup$
    – 5xum
    Mar 26 at 7:52










  • $begingroup$
    Can explain how you jumped from $|B| leq |A|$ to $|Acup B|=|A|$?
    $endgroup$
    – Kavi Rama Murthy
    Mar 26 at 7:54










  • $begingroup$
    Probably quite a few more questions of this nature are to be found elsewhere around the site.
    $endgroup$
    – Asaf Karagila
    Mar 27 at 10:51















0












$begingroup$



This question already has an answer here:



  • Cardinality of the union of infinite and countable sets

    2 answers



  • Let the set S be infinite, and the set T countably infinite. Show that S and S U T have the same cardinality

    1 answer



Can someone please verify if my proof is correct, and add to it if it isn't?



Since we're given that $B$ is countable, we know $B$ is either finite or has the same cardinality as, say, the set $mathbbQ$. So $vert B vert = aleph_0$.



Similarly, if $A$ is infinite, then we know the result $aleph_0 leq vert A vert$.



This implies that $ vert B vert leq vert A vert$.



Then $vert A cup B vert = vert A vert$.



Am I missing something here?










share|cite|improve this question











$endgroup$



marked as duplicate by Asaf Karagila elementary-set-theory
Users with the  elementary-set-theory badge can single-handedly close elementary-set-theory questions as duplicates and reopen them as needed.

StackExchange.ready(function()
if (StackExchange.options.isMobile) return;

$('.dupe-hammer-message-hover:not(.hover-bound)').each(function()
var $hover = $(this).addClass('hover-bound'),
$msg = $hover.siblings('.dupe-hammer-message');

$hover.hover(
function()
$hover.showInfoMessage('',
messageElement: $msg.clone().show(),
transient: false,
position: my: 'bottom left', at: 'top center', offsetTop: -7 ,
dismissable: false,
relativeToBody: true
);
,
function()
StackExchange.helpers.removeMessages();

);
);
);
Mar 27 at 10:50


This question has been asked before and already has an answer. If those answers do not fully address your question, please ask a new question.

















  • $begingroup$
    Please include the original question in the body of your question.
    $endgroup$
    – coreyman317
    Mar 26 at 7:51










  • $begingroup$
    The answer to this question will be practically the same as the answer to math.stackexchange.com/questions/3162848/… .
    $endgroup$
    – 5xum
    Mar 26 at 7:52










  • $begingroup$
    Can explain how you jumped from $|B| leq |A|$ to $|Acup B|=|A|$?
    $endgroup$
    – Kavi Rama Murthy
    Mar 26 at 7:54










  • $begingroup$
    Probably quite a few more questions of this nature are to be found elsewhere around the site.
    $endgroup$
    – Asaf Karagila
    Mar 27 at 10:51













0












0








0


0



$begingroup$



This question already has an answer here:



  • Cardinality of the union of infinite and countable sets

    2 answers



  • Let the set S be infinite, and the set T countably infinite. Show that S and S U T have the same cardinality

    1 answer



Can someone please verify if my proof is correct, and add to it if it isn't?



Since we're given that $B$ is countable, we know $B$ is either finite or has the same cardinality as, say, the set $mathbbQ$. So $vert B vert = aleph_0$.



Similarly, if $A$ is infinite, then we know the result $aleph_0 leq vert A vert$.



This implies that $ vert B vert leq vert A vert$.



Then $vert A cup B vert = vert A vert$.



Am I missing something here?










share|cite|improve this question











$endgroup$





This question already has an answer here:



  • Cardinality of the union of infinite and countable sets

    2 answers



  • Let the set S be infinite, and the set T countably infinite. Show that S and S U T have the same cardinality

    1 answer



Can someone please verify if my proof is correct, and add to it if it isn't?



Since we're given that $B$ is countable, we know $B$ is either finite or has the same cardinality as, say, the set $mathbbQ$. So $vert B vert = aleph_0$.



Similarly, if $A$ is infinite, then we know the result $aleph_0 leq vert A vert$.



This implies that $ vert B vert leq vert A vert$.



Then $vert A cup B vert = vert A vert$.



Am I missing something here?





This question already has an answer here:



  • Cardinality of the union of infinite and countable sets

    2 answers



  • Let the set S be infinite, and the set T countably infinite. Show that S and S U T have the same cardinality

    1 answer







proof-verification elementary-set-theory proof-writing alternative-proof






share|cite|improve this question















share|cite|improve this question













share|cite|improve this question




share|cite|improve this question








edited Mar 26 at 7:51









5xum

92.7k394162




92.7k394162










asked Mar 26 at 7:49









Yousaf05Yousaf05

414




414




marked as duplicate by Asaf Karagila elementary-set-theory
Users with the  elementary-set-theory badge can single-handedly close elementary-set-theory questions as duplicates and reopen them as needed.

StackExchange.ready(function()
if (StackExchange.options.isMobile) return;

$('.dupe-hammer-message-hover:not(.hover-bound)').each(function()
var $hover = $(this).addClass('hover-bound'),
$msg = $hover.siblings('.dupe-hammer-message');

$hover.hover(
function()
$hover.showInfoMessage('',
messageElement: $msg.clone().show(),
transient: false,
position: my: 'bottom left', at: 'top center', offsetTop: -7 ,
dismissable: false,
relativeToBody: true
);
,
function()
StackExchange.helpers.removeMessages();

);
);
);
Mar 27 at 10:50


This question has been asked before and already has an answer. If those answers do not fully address your question, please ask a new question.









marked as duplicate by Asaf Karagila elementary-set-theory
Users with the  elementary-set-theory badge can single-handedly close elementary-set-theory questions as duplicates and reopen them as needed.

StackExchange.ready(function()
if (StackExchange.options.isMobile) return;

$('.dupe-hammer-message-hover:not(.hover-bound)').each(function()
var $hover = $(this).addClass('hover-bound'),
$msg = $hover.siblings('.dupe-hammer-message');

$hover.hover(
function()
$hover.showInfoMessage('',
messageElement: $msg.clone().show(),
transient: false,
position: my: 'bottom left', at: 'top center', offsetTop: -7 ,
dismissable: false,
relativeToBody: true
);
,
function()
StackExchange.helpers.removeMessages();

);
);
);
Mar 27 at 10:50


This question has been asked before and already has an answer. If those answers do not fully address your question, please ask a new question.













  • $begingroup$
    Please include the original question in the body of your question.
    $endgroup$
    – coreyman317
    Mar 26 at 7:51










  • $begingroup$
    The answer to this question will be practically the same as the answer to math.stackexchange.com/questions/3162848/… .
    $endgroup$
    – 5xum
    Mar 26 at 7:52










  • $begingroup$
    Can explain how you jumped from $|B| leq |A|$ to $|Acup B|=|A|$?
    $endgroup$
    – Kavi Rama Murthy
    Mar 26 at 7:54










  • $begingroup$
    Probably quite a few more questions of this nature are to be found elsewhere around the site.
    $endgroup$
    – Asaf Karagila
    Mar 27 at 10:51
















  • $begingroup$
    Please include the original question in the body of your question.
    $endgroup$
    – coreyman317
    Mar 26 at 7:51










  • $begingroup$
    The answer to this question will be practically the same as the answer to math.stackexchange.com/questions/3162848/… .
    $endgroup$
    – 5xum
    Mar 26 at 7:52










  • $begingroup$
    Can explain how you jumped from $|B| leq |A|$ to $|Acup B|=|A|$?
    $endgroup$
    – Kavi Rama Murthy
    Mar 26 at 7:54










  • $begingroup$
    Probably quite a few more questions of this nature are to be found elsewhere around the site.
    $endgroup$
    – Asaf Karagila
    Mar 27 at 10:51















$begingroup$
Please include the original question in the body of your question.
$endgroup$
– coreyman317
Mar 26 at 7:51




$begingroup$
Please include the original question in the body of your question.
$endgroup$
– coreyman317
Mar 26 at 7:51












$begingroup$
The answer to this question will be practically the same as the answer to math.stackexchange.com/questions/3162848/… .
$endgroup$
– 5xum
Mar 26 at 7:52




$begingroup$
The answer to this question will be practically the same as the answer to math.stackexchange.com/questions/3162848/… .
$endgroup$
– 5xum
Mar 26 at 7:52












$begingroup$
Can explain how you jumped from $|B| leq |A|$ to $|Acup B|=|A|$?
$endgroup$
– Kavi Rama Murthy
Mar 26 at 7:54




$begingroup$
Can explain how you jumped from $|B| leq |A|$ to $|Acup B|=|A|$?
$endgroup$
– Kavi Rama Murthy
Mar 26 at 7:54












$begingroup$
Probably quite a few more questions of this nature are to be found elsewhere around the site.
$endgroup$
– Asaf Karagila
Mar 27 at 10:51




$begingroup$
Probably quite a few more questions of this nature are to be found elsewhere around the site.
$endgroup$
– Asaf Karagila
Mar 27 at 10:51










2 Answers
2






active

oldest

votes


















1












$begingroup$

I am not at all expert in large cardinals, but as far as my understanding goes, once you have proved $|B| = aleph_0$, you then have (assuming that $A$ and $B$ are disjoint) $|Acup B| = |A| + |B| = |A| + aleph_0$, and the arithmetic cardinals is such that adding $aleph_0$ to any infinite cardinal does not change said cardinal.



That being said, this "arithmetic" property is no less than what you are asked to show here, and I think the point of the question is to make you prove that. As @5xum suggested in his comment, you can define an injection $A to Acup B$ together with a surjection $Ato Acup B$ and use the Cantor-Bernstein theorem.



To get a feel for how to do that, I would suggest to try to understand how a bijection between $mathbbN$ and $mathbbN^2$ works.
Also note that without loss of generality, you can assume $B = mathbbQ$ ou $mathbbN$. Indeed since $|mathbbQ| = |mathbbN| = aleph_0$, you have bijections $BtomathbbN$ and $BtomathbbQ$. So if you get a bijection $Ato Acup mathbbN$, you can then use the fact that the composition of bijections is a bijection.
Finally you can assume that $mathbbNsubset A$ (more precisely, there is a subset of $A$ which is in bijection with $mathbbN$). This should help you mimic the construction of the case $mathbbN^2$






share|cite|improve this answer









$endgroup$












  • $begingroup$
    The first sentence is self evident. Since large cardinals are a certain type of set theoretic axioms which have a technical meaning to them.
    $endgroup$
    – Asaf Karagila
    Mar 27 at 10:49










  • $begingroup$
    I must have seen this at some point and forgot about it, hence my confusion thinking large cardinals were infinite cardinals in ZFC...
    $endgroup$
    – Thibaut Benjamin
    Mar 27 at 10:54


















0












$begingroup$

You are missing something, yes. You only wrote down the following relations:




  • $|B|=aleph_0$, a relation that is not even true! There is nothing in the question that prevents $B$ from being a finite set.

  • $|B|leq |A|$

  • $aleph_0leq |A|$

There is nothing here for you to conclude that $|Acup B|=|A|$. For that, you would have to prove that $|A|leq |Acup B|$ (this should be easy) and that $|Acup B|leq |A|$ (this one should prove a little harder, as you need a surjection from $A$ to $Acup B$ to prove this inequality).






share|cite|improve this answer









$endgroup$












  • $begingroup$
    Thanks! I see but what if we are given $B$ is a countably infinite set. Would that be enough to say $|B|=aleph_0$?
    $endgroup$
    – Yousaf05
    Mar 26 at 8:09











  • $begingroup$
    @Yousaf05 Since $|mathbb Q|=aleph_0$, yes, of course. But that still leaves you with a lot of work to do.
    $endgroup$
    – 5xum
    Mar 26 at 8:10

















2 Answers
2






active

oldest

votes








2 Answers
2






active

oldest

votes









active

oldest

votes






active

oldest

votes









1












$begingroup$

I am not at all expert in large cardinals, but as far as my understanding goes, once you have proved $|B| = aleph_0$, you then have (assuming that $A$ and $B$ are disjoint) $|Acup B| = |A| + |B| = |A| + aleph_0$, and the arithmetic cardinals is such that adding $aleph_0$ to any infinite cardinal does not change said cardinal.



That being said, this "arithmetic" property is no less than what you are asked to show here, and I think the point of the question is to make you prove that. As @5xum suggested in his comment, you can define an injection $A to Acup B$ together with a surjection $Ato Acup B$ and use the Cantor-Bernstein theorem.



To get a feel for how to do that, I would suggest to try to understand how a bijection between $mathbbN$ and $mathbbN^2$ works.
Also note that without loss of generality, you can assume $B = mathbbQ$ ou $mathbbN$. Indeed since $|mathbbQ| = |mathbbN| = aleph_0$, you have bijections $BtomathbbN$ and $BtomathbbQ$. So if you get a bijection $Ato Acup mathbbN$, you can then use the fact that the composition of bijections is a bijection.
Finally you can assume that $mathbbNsubset A$ (more precisely, there is a subset of $A$ which is in bijection with $mathbbN$). This should help you mimic the construction of the case $mathbbN^2$






share|cite|improve this answer









$endgroup$












  • $begingroup$
    The first sentence is self evident. Since large cardinals are a certain type of set theoretic axioms which have a technical meaning to them.
    $endgroup$
    – Asaf Karagila
    Mar 27 at 10:49










  • $begingroup$
    I must have seen this at some point and forgot about it, hence my confusion thinking large cardinals were infinite cardinals in ZFC...
    $endgroup$
    – Thibaut Benjamin
    Mar 27 at 10:54















1












$begingroup$

I am not at all expert in large cardinals, but as far as my understanding goes, once you have proved $|B| = aleph_0$, you then have (assuming that $A$ and $B$ are disjoint) $|Acup B| = |A| + |B| = |A| + aleph_0$, and the arithmetic cardinals is such that adding $aleph_0$ to any infinite cardinal does not change said cardinal.



That being said, this "arithmetic" property is no less than what you are asked to show here, and I think the point of the question is to make you prove that. As @5xum suggested in his comment, you can define an injection $A to Acup B$ together with a surjection $Ato Acup B$ and use the Cantor-Bernstein theorem.



To get a feel for how to do that, I would suggest to try to understand how a bijection between $mathbbN$ and $mathbbN^2$ works.
Also note that without loss of generality, you can assume $B = mathbbQ$ ou $mathbbN$. Indeed since $|mathbbQ| = |mathbbN| = aleph_0$, you have bijections $BtomathbbN$ and $BtomathbbQ$. So if you get a bijection $Ato Acup mathbbN$, you can then use the fact that the composition of bijections is a bijection.
Finally you can assume that $mathbbNsubset A$ (more precisely, there is a subset of $A$ which is in bijection with $mathbbN$). This should help you mimic the construction of the case $mathbbN^2$






share|cite|improve this answer









$endgroup$












  • $begingroup$
    The first sentence is self evident. Since large cardinals are a certain type of set theoretic axioms which have a technical meaning to them.
    $endgroup$
    – Asaf Karagila
    Mar 27 at 10:49










  • $begingroup$
    I must have seen this at some point and forgot about it, hence my confusion thinking large cardinals were infinite cardinals in ZFC...
    $endgroup$
    – Thibaut Benjamin
    Mar 27 at 10:54













1












1








1





$begingroup$

I am not at all expert in large cardinals, but as far as my understanding goes, once you have proved $|B| = aleph_0$, you then have (assuming that $A$ and $B$ are disjoint) $|Acup B| = |A| + |B| = |A| + aleph_0$, and the arithmetic cardinals is such that adding $aleph_0$ to any infinite cardinal does not change said cardinal.



That being said, this "arithmetic" property is no less than what you are asked to show here, and I think the point of the question is to make you prove that. As @5xum suggested in his comment, you can define an injection $A to Acup B$ together with a surjection $Ato Acup B$ and use the Cantor-Bernstein theorem.



To get a feel for how to do that, I would suggest to try to understand how a bijection between $mathbbN$ and $mathbbN^2$ works.
Also note that without loss of generality, you can assume $B = mathbbQ$ ou $mathbbN$. Indeed since $|mathbbQ| = |mathbbN| = aleph_0$, you have bijections $BtomathbbN$ and $BtomathbbQ$. So if you get a bijection $Ato Acup mathbbN$, you can then use the fact that the composition of bijections is a bijection.
Finally you can assume that $mathbbNsubset A$ (more precisely, there is a subset of $A$ which is in bijection with $mathbbN$). This should help you mimic the construction of the case $mathbbN^2$






share|cite|improve this answer









$endgroup$



I am not at all expert in large cardinals, but as far as my understanding goes, once you have proved $|B| = aleph_0$, you then have (assuming that $A$ and $B$ are disjoint) $|Acup B| = |A| + |B| = |A| + aleph_0$, and the arithmetic cardinals is such that adding $aleph_0$ to any infinite cardinal does not change said cardinal.



That being said, this "arithmetic" property is no less than what you are asked to show here, and I think the point of the question is to make you prove that. As @5xum suggested in his comment, you can define an injection $A to Acup B$ together with a surjection $Ato Acup B$ and use the Cantor-Bernstein theorem.



To get a feel for how to do that, I would suggest to try to understand how a bijection between $mathbbN$ and $mathbbN^2$ works.
Also note that without loss of generality, you can assume $B = mathbbQ$ ou $mathbbN$. Indeed since $|mathbbQ| = |mathbbN| = aleph_0$, you have bijections $BtomathbbN$ and $BtomathbbQ$. So if you get a bijection $Ato Acup mathbbN$, you can then use the fact that the composition of bijections is a bijection.
Finally you can assume that $mathbbNsubset A$ (more precisely, there is a subset of $A$ which is in bijection with $mathbbN$). This should help you mimic the construction of the case $mathbbN^2$







share|cite|improve this answer












share|cite|improve this answer



share|cite|improve this answer










answered Mar 26 at 10:19









Thibaut BenjaminThibaut Benjamin

38119




38119











  • $begingroup$
    The first sentence is self evident. Since large cardinals are a certain type of set theoretic axioms which have a technical meaning to them.
    $endgroup$
    – Asaf Karagila
    Mar 27 at 10:49










  • $begingroup$
    I must have seen this at some point and forgot about it, hence my confusion thinking large cardinals were infinite cardinals in ZFC...
    $endgroup$
    – Thibaut Benjamin
    Mar 27 at 10:54
















  • $begingroup$
    The first sentence is self evident. Since large cardinals are a certain type of set theoretic axioms which have a technical meaning to them.
    $endgroup$
    – Asaf Karagila
    Mar 27 at 10:49










  • $begingroup$
    I must have seen this at some point and forgot about it, hence my confusion thinking large cardinals were infinite cardinals in ZFC...
    $endgroup$
    – Thibaut Benjamin
    Mar 27 at 10:54















$begingroup$
The first sentence is self evident. Since large cardinals are a certain type of set theoretic axioms which have a technical meaning to them.
$endgroup$
– Asaf Karagila
Mar 27 at 10:49




$begingroup$
The first sentence is self evident. Since large cardinals are a certain type of set theoretic axioms which have a technical meaning to them.
$endgroup$
– Asaf Karagila
Mar 27 at 10:49












$begingroup$
I must have seen this at some point and forgot about it, hence my confusion thinking large cardinals were infinite cardinals in ZFC...
$endgroup$
– Thibaut Benjamin
Mar 27 at 10:54




$begingroup$
I must have seen this at some point and forgot about it, hence my confusion thinking large cardinals were infinite cardinals in ZFC...
$endgroup$
– Thibaut Benjamin
Mar 27 at 10:54











0












$begingroup$

You are missing something, yes. You only wrote down the following relations:




  • $|B|=aleph_0$, a relation that is not even true! There is nothing in the question that prevents $B$ from being a finite set.

  • $|B|leq |A|$

  • $aleph_0leq |A|$

There is nothing here for you to conclude that $|Acup B|=|A|$. For that, you would have to prove that $|A|leq |Acup B|$ (this should be easy) and that $|Acup B|leq |A|$ (this one should prove a little harder, as you need a surjection from $A$ to $Acup B$ to prove this inequality).






share|cite|improve this answer









$endgroup$












  • $begingroup$
    Thanks! I see but what if we are given $B$ is a countably infinite set. Would that be enough to say $|B|=aleph_0$?
    $endgroup$
    – Yousaf05
    Mar 26 at 8:09











  • $begingroup$
    @Yousaf05 Since $|mathbb Q|=aleph_0$, yes, of course. But that still leaves you with a lot of work to do.
    $endgroup$
    – 5xum
    Mar 26 at 8:10















0












$begingroup$

You are missing something, yes. You only wrote down the following relations:




  • $|B|=aleph_0$, a relation that is not even true! There is nothing in the question that prevents $B$ from being a finite set.

  • $|B|leq |A|$

  • $aleph_0leq |A|$

There is nothing here for you to conclude that $|Acup B|=|A|$. For that, you would have to prove that $|A|leq |Acup B|$ (this should be easy) and that $|Acup B|leq |A|$ (this one should prove a little harder, as you need a surjection from $A$ to $Acup B$ to prove this inequality).






share|cite|improve this answer









$endgroup$












  • $begingroup$
    Thanks! I see but what if we are given $B$ is a countably infinite set. Would that be enough to say $|B|=aleph_0$?
    $endgroup$
    – Yousaf05
    Mar 26 at 8:09











  • $begingroup$
    @Yousaf05 Since $|mathbb Q|=aleph_0$, yes, of course. But that still leaves you with a lot of work to do.
    $endgroup$
    – 5xum
    Mar 26 at 8:10













0












0








0





$begingroup$

You are missing something, yes. You only wrote down the following relations:




  • $|B|=aleph_0$, a relation that is not even true! There is nothing in the question that prevents $B$ from being a finite set.

  • $|B|leq |A|$

  • $aleph_0leq |A|$

There is nothing here for you to conclude that $|Acup B|=|A|$. For that, you would have to prove that $|A|leq |Acup B|$ (this should be easy) and that $|Acup B|leq |A|$ (this one should prove a little harder, as you need a surjection from $A$ to $Acup B$ to prove this inequality).






share|cite|improve this answer









$endgroup$



You are missing something, yes. You only wrote down the following relations:




  • $|B|=aleph_0$, a relation that is not even true! There is nothing in the question that prevents $B$ from being a finite set.

  • $|B|leq |A|$

  • $aleph_0leq |A|$

There is nothing here for you to conclude that $|Acup B|=|A|$. For that, you would have to prove that $|A|leq |Acup B|$ (this should be easy) and that $|Acup B|leq |A|$ (this one should prove a little harder, as you need a surjection from $A$ to $Acup B$ to prove this inequality).







share|cite|improve this answer












share|cite|improve this answer



share|cite|improve this answer










answered Mar 26 at 7:58









5xum5xum

92.7k394162




92.7k394162











  • $begingroup$
    Thanks! I see but what if we are given $B$ is a countably infinite set. Would that be enough to say $|B|=aleph_0$?
    $endgroup$
    – Yousaf05
    Mar 26 at 8:09











  • $begingroup$
    @Yousaf05 Since $|mathbb Q|=aleph_0$, yes, of course. But that still leaves you with a lot of work to do.
    $endgroup$
    – 5xum
    Mar 26 at 8:10
















  • $begingroup$
    Thanks! I see but what if we are given $B$ is a countably infinite set. Would that be enough to say $|B|=aleph_0$?
    $endgroup$
    – Yousaf05
    Mar 26 at 8:09











  • $begingroup$
    @Yousaf05 Since $|mathbb Q|=aleph_0$, yes, of course. But that still leaves you with a lot of work to do.
    $endgroup$
    – 5xum
    Mar 26 at 8:10















$begingroup$
Thanks! I see but what if we are given $B$ is a countably infinite set. Would that be enough to say $|B|=aleph_0$?
$endgroup$
– Yousaf05
Mar 26 at 8:09





$begingroup$
Thanks! I see but what if we are given $B$ is a countably infinite set. Would that be enough to say $|B|=aleph_0$?
$endgroup$
– Yousaf05
Mar 26 at 8:09













$begingroup$
@Yousaf05 Since $|mathbb Q|=aleph_0$, yes, of course. But that still leaves you with a lot of work to do.
$endgroup$
– 5xum
Mar 26 at 8:10




$begingroup$
@Yousaf05 Since $|mathbb Q|=aleph_0$, yes, of course. But that still leaves you with a lot of work to do.
$endgroup$
– 5xum
Mar 26 at 8:10



Popular posts from this blog

Lowndes Grove History Architecture References Navigation menu32°48′6″N 79°57′58″W / 32.80167°N 79.96611°W / 32.80167; -79.9661132°48′6″N 79°57′58″W / 32.80167°N 79.96611°W / 32.80167; -79.9661178002500"National Register Information System"Historic houses of South Carolina"Lowndes Grove""+32° 48' 6.00", −79° 57' 58.00""Lowndes Grove, Charleston County (260 St. Margaret St., Charleston)""Lowndes Grove"The Charleston ExpositionIt Happened in South Carolina"Lowndes Grove (House), Saint Margaret Street & Sixth Avenue, Charleston, Charleston County, SC(Photographs)"Plantations of the Carolina Low Countrye

random experiment with two different functions on unit interval Announcing the arrival of Valued Associate #679: Cesar Manara Planned maintenance scheduled April 23, 2019 at 00:00UTC (8:00pm US/Eastern)Random variable and probability space notionsRandom Walk with EdgesFinding functions where the increase over a random interval is Poisson distributedNumber of days until dayCan an observed event in fact be of zero probability?Unit random processmodels of coins and uniform distributionHow to get the number of successes given $n$ trials , probability $P$ and a random variable $X$Absorbing Markov chain in a computer. Is “almost every” turned into always convergence in computer executions?Stopped random walk is not uniformly integrable

How should I support this large drywall patch? Planned maintenance scheduled April 23, 2019 at 00:00UTC (8:00pm US/Eastern) Announcing the arrival of Valued Associate #679: Cesar Manara Unicorn Meta Zoo #1: Why another podcast?How do I cover large gaps in drywall?How do I keep drywall around a patch from crumbling?Can I glue a second layer of drywall?How to patch long strip on drywall?Large drywall patch: how to avoid bulging seams?Drywall Mesh Patch vs. Bulge? To remove or not to remove?How to fix this drywall job?Prep drywall before backsplashWhat's the best way to fix this horrible drywall patch job?Drywall patching using 3M Patch Plus Primer